LSAT and Law School Admissions Forum

Get expert LSAT preparation and law school admissions advice from PowerScore Test Preparation.

User avatar
 Dave Killoran
PowerScore Staff
  • PowerScore Staff
  • Posts: 5852
  • Joined: Mar 25, 2011
|
#47494
Complete Question Explanation
(The complete setup for this game can be found here: lsat/viewtopic.php?t=17376)

The correct answer choice is (C)

Answer choices (A), (B), and (E) can all occur when F is in chair 1. Answer choices (B), (D), and (E) can occur when F is seat 3.

Only answer choice (C) cannot occur, and thus (C) is the correct answer. For both H and J to sit west of F, F would have to be in chair 7, and that placement would create a violation of the last rule.

Get the most out of your LSAT Prep Plus subscription.

Analyze and track your performance with our Testing and Analytics Package.